Como responder a perguntas Exceto argumentação no LSAT

Muitos reforçar / enfraquecem perguntas sobre a LSAT não pedem a escolha que melhor se fortalece (ou enfraquece, ou suportes, ou mina) o argumento, mas sim para a escolha que não faz fazer uma dessas coisas. Estas perguntas terminar na palavra extremamente importante EXCETO. É sempre em maiúsculas, para que você não pode perder.

Estas perguntas parecido com este:

  • Cada um dos seguintes, se for verdade, sustenta o argumento acima, exceto:

  • Cada um dos seguintes, se for verdade, enfraqueceria o argumento do comentarista, EXCETO:

  • Cada um dos seguintes, se for verdade, suporta a hipótese do físico, EXCETO:

  • Cada um dos seguintes, se for verdade, enfraquece o argumento EXCETO:

  • Cada um dos seguintes, se for verdade, reforça o argumento EXCETO:

Eles também podem ter esta aparência:

  • Qual dos seguintes, se todas elas são verdadeiras, é MENOS útil para estabelecer que a conclusão acima é desenhada corretamente?

  • Qual dos seguintes, se todas elas são verdadeiras, é menos eficazes a minar o argumento do político?

Eles ainda estão reforçar / enfraquecer perguntas, mas em vez de encontrar uma resposta para reforçar / enfraquecer / support / minar, você quer encontrar quatro.

Como sempre, leu pela primeira vez a questão cuidadosamente. Estas questões não são o seu padrão reforçar / enfraquecer perguntas. Em vez disso, eles oferecem quatro opções que fortalecer ou enfraquecer a conclusão, e um que não. Você não necessariamente tem que encontrar uma resposta que fortalece ou enfraquece a conclusion- você só quer uma resposta que não faz o que os outros quatro fazer.

Por exemplo, se a questão lê # 147 cada um dos seguintes, se for verdade, enfraquece o argumento EXCETO: # 148- a resposta correta não significa necessariamente fortalecer o argumento, mas ele definitivamente não enfraquecê-lo.

Aqui está um exemplo de uma questão que tem quatro respostas que enfraquecem o argumento e que não faz:

Alguns trabalhadores em fábricas de pipoca de microondas ter contraído uma doença pulmonar rara. Os especialistas têm ligado esta doença a uma substância química utilizada no processo de mistura de pipoca e aromas. Os consumidores devem, portanto, parar de comprar e comer todos os tipos de pipoca de microondas para evitar o risco de contrair esta doença pulmonar.

Cada um dos seguintes, se for verdade, enfraquece o argumento EXCETO:

  • (A) A doença do pulmão é causada por serem expostos ao produto químico para muitas horas de cada vez ao longo de um período de anos.

  • (B) O produto químico só se torna tóxico quando é realizada a uma temperatura muito mais elevada do que a alcançada por pipocas num forno de microondas.

  • (C) A doença pulmonar só foi encontrada em trabalhadores que lidavam com o sabor Cajun Spice de pipoca.

  • (D) Em 20 anos de consumo de pipoca de microondas generalizada, nenhum consumidor já contraiu essa doença pulmonar rara.

  • (E) O EPA ainda não tenha feito qualquer investigação para determinar se o produto químico que provoca a doença pulmonar está presente no vapor e ar que sai de um saco de pipoca quando ele é aberto.

Da leitura da questão em primeiro lugar, você sabe que quer uma resposta que não enfraquece o argumento. A conclusão é que os consumidores devem parar de comer todos os tipos de pipoca de microondas para evitar contrair uma doença. A evidência para essa conclusão é que um produto químico na pipoca de microondas causou esta doença em funcionários pipoca fábrica.

O que enfraquece o argumento? Qualquer evidência que faz com que os consumidores parecem seguros. Se você puder provar que a doença é de alguma forma restrita aos trabalhadores da fábrica, você pode garantir aos consumidores que eles podem comer pipoca de microondas com impunidade. O que não enfraquece a conclusão? Qualquer evidência indicando que os consumidores podem pegar a doença de comer pipoca de microondas ou qualquer evidência de que é de relevância claro ou simplesmente off-topic.

  • A escolha (a) enfraquece o argumento porque a maioria dos consumidores não estão expostas à substância em questão por longos períodos de tempo.

  • Choice (B) enfraquece o argumento porque os consumidores não estão mantendo sua pipoca quente o suficiente.

  • Choice (C) enfraquece a sugestão de que os consumidores devem evitar todos os tipos de pipoca de microondas, embora possivelmente não a sugestão de que eles devem evitar Cajun Spice sabor.

  • Choice (D) enfraquece a sugestão, porque torna o risco para os consumidores parecem quase insignificante.

  • Choice (E) não enfraquece a argument- se a EPA ainda não tenha feito qualquer investigação à presença do produto químico no vapor pipoca, então a química poderia muito bem ser à espreita lá, pronto para adoecer consumidores infelizes.

Choice (E) não especialmente reforçar o argumento, mas não enfraquecê-lo, tampouco. Choice (E) é a resposta correta.

menu